what type of question is this?
?
Skyler-Simon on July 13, 2020
  • October 2013 LSAT
  • SEC1
  • Q15
1
Reply
Answer B vs D
Hello! I understand why B is correct, but I am not sure why D is incorrect. Coule you please expl...
Ame15 on July 12, 2020
  • October 2013 LSAT
  • SEC1
  • Q13
1
Reply
A vs. E
I had an issue with this question. I get that A doesn't say anything about the bill being favored...
avif on June 15, 2020
  • October 2013 LSAT
  • SEC1
  • Q23
2
Replies
Why B over E?
I am not co pletlry certain on this one
zgnewquist on June 6, 2020
  • October 2013 LSAT
  • SEC1
  • Q12
3
Replies
Answer choice D
Can someone explain why answer choice D is incorrect and B is correct? I was choosing between the...
shafieiava on April 22, 2020
  • October 2013 LSAT
  • SEC1
  • Q8
1
Reply
Answer choices B and E
Can someone explain the difference between answer choices B and E? It seems to me they are saying...
shafieiava on April 22, 2020
  • October 2013 LSAT
  • SEC1
  • Q23
1
Reply
Please Explain
Why is the answer D compared to of E
rpark on December 28, 2019
  • October 2013 LSAT
  • SEC1
  • Q14
1
Reply
Help!
Can someone please explain this question and the answer choices? I'm a bit confused on how to rea...
Lauren-Au on December 27, 2019
  • October 2013 LSAT
  • SEC1
  • Q23
1
Reply
Question stem
I'm confused about what do look for in the AC. I know we have to apply the principle to the probl...
SeonAh-Lee on December 27, 2019
  • October 2013 LSAT
  • SEC1
  • Q6
2
Replies
Why is A correct? Why is B incorrect?
Thanks
Ryan-Mahabir on December 27, 2019
  • October 2013 LSAT
  • SEC1
  • Q1
1
Reply
Why is the answer C?
Can someone please explain this? How are some physicians mistaken about the degree of influence?
lerondagates on December 8, 2019
  • October 2013 LSAT
  • SEC1
  • Q22
4
Replies
Why is A correct? Why is E incorrect?
Thanks
Ryan-Mahabir on November 1, 2019
  • October 2013 LSAT
  • SEC1
  • Q18
1
Reply
Argument Evaluation
In the video, you have explained that we can consider the extremes to find the answer. How can we...
SeonAh-Lee on October 15, 2019
  • October 2013 LSAT
  • SEC1
  • Q11
1
Reply
Why is A wrong?
I couldn't choose between A and C. Please explain it.
SeonAh-Lee on October 7, 2019
  • October 2013 LSAT
  • SEC1
  • Q19
1
Reply
Need help!
I'm totally lost with this question. Please help!
SeonAh-Lee on October 7, 2019
  • October 2013 LSAT
  • SEC1
  • Q23
1
Reply
Why is E incorrect?
Thanks
Ryan-Mahabir on October 7, 2019
  • October 2013 LSAT
  • SEC1
  • Q6
1
Reply
Correct Answer
What is the difference between answer D and C? They seem to both offer valid ways of rectifying t...
Lizzie-Annerino on August 14, 2019
  • October 2013 LSAT
  • SEC1
  • Q6
1
Reply
Can you please explain?
Can someone explain why D is the correct answer and not C?
Jaimee-Salgado on October 17, 2018
  • October 2013 LSAT
  • SEC1
  • Q21
1
Reply
Question
Can you please explain why A is the correct answer choice?
Remy on September 29, 2015
  • October 2013 LSAT
  • SEC1
  • Q15
1
Reply
I don't understand
Why is a wrong?
pkhenry on June 5, 2015
  • October 2013 LSAT
  • SEC1
  • Q16
1
Reply